Answered step by step
Verified Expert Solution
Link Copied!

Question

1 Approved Answer

Up Down Left Right Net sales revenue $ 1,064,000 $ (d) $ 525,000 $ (j) Variable Costs (a) 104,000 315,000 95,700 Fixed Costs (b) 168,000

Up

Down

Left

Right

Net sales revenue

$ 1,064,000

$ (d)

$ 525,000

$ (j)

Variable Costs

(a)

104,000

315,000

95,700

Fixed Costs

(b)

168,000

200,000

(k)

Operating Income (loss)

89,500

$ (e)

$ (g)

$ 49,800

Units Sold

140,000

16,000

(h)

(I)

Contribution Margin per unit

3.80

$ (f)

$ 70.00

$ 11.00

Contribution Margin Ratio

( c)

80%

(i)

40%

Round the contribution margin per unit to the nearest cent. Use a minus sign or parentheses to enter an operating loss.)

Save Accounting Table...

+

Copy to Clipboard...

+

Up

Net Sales Revenue

$1,064,000

Variable Costs

________

Fixed Costs

________

Operating Income (Loss)

$89,500

Units Sold

140,000

Contribution Margin per Unit

$3.80

Contribution Margin Ratio _

%

Step by Step Solution

There are 3 Steps involved in it

Step: 1

blur-text-image

Get Instant Access to Expert-Tailored Solutions

See step-by-step solutions with expert insights and AI powered tools for academic success

Step: 2

blur-text-image

Step: 3

blur-text-image

Ace Your Homework with AI

Get the answers you need in no time with our AI-driven, step-by-step assistance

Get Started

Recommended Textbook for

How To Secure And Audit Oracle 10g And 11g

Authors: Ron Ben-Natan, Brian E. White, Paul R. Garvey

1st Edition

1420084127, 978-1420084122

More Books

Students also viewed these Accounting questions

Question

What are the attributes of a technical decision?

Answered: 1 week ago

Question

How do the two components of this theory work together?

Answered: 1 week ago

Question

Distinguish between formal and informal reports.

Answered: 1 week ago